How did Ramanujan find $\sum_{n=0}^\infty (-1)^n\frac{(1/2)_n(1/4)_n(3/4)_n}{n!^3}\frac{644n+41}{25920^n}=\frac{288\sqrt{5}}{5\pi}?$

definite integralselliptic integralshypergeometric functionpisequences-and-series

The formula
$$\sum_{n=0}^\infty (-1)^n\frac{(1/2)_n(1/4)_n(3/4)_n}{n!^3}\frac{644n+41}{25920^n}=\frac{288\sqrt{5}}{5\pi}$$
(in older notation) appears as eq. 38 in Ramanujan's paper Modular equations and approximations to $\pi$; $(a)_n$ is the Pochhammer symbol.

But – the formula is like an exercise for the reader. Allegedly, it can be deduced from the theory of modular equations, but how exactly?

It should be somehow possible to prove the formula from Clausen's formulas and representations of
$$25P(q^{50})-P(q^2)$$
where
$$P(q)=1-24\sum_{k=1}^\infty \frac{kq^k}{1-q^k}$$
with $|q|\lt 1$. The hypergeometric representation of Ramanujan's formula is known to be
$$41\,_3F_2\left(\frac14,\frac12,\frac34,1,1,-\frac{1}{25920}\right)-\frac{161}{69120}\, _3F_2\left(\frac54,\frac32,\frac74,2,2,-\frac{1}{25920}\right)=\frac{288\sqrt{5}}{5\pi}.$$

What I tried

Let
$$K(x)=\int_0^{\pi/2}\dfrac{dt}{\sqrt{1-x\sin^2 t}}$$
and
$$E(x)=\int_0^{\pi/2}\sqrt{1-x\sin^2 t}\,dt$$
be the elliptic integrals.
Using the hypergeometric differential equation, the problem at hand shoud then be reducible to
$$K\left(\dfrac{1}{2}-6\sqrt{-360+161\sqrt{5}}\right)\left(2E\left(\dfrac{1}{2}-6\sqrt{-360+161\sqrt{5}}\right)-\left(1+\sqrt{-840+376\sqrt{5}}\right)K\left(\dfrac{1}{2}-6\sqrt{-360+161\sqrt{5}}\right)\right)=\frac{\pi}{10}.$$

Now I recognized the argument of the elliptic integrals as a special value of the modular lambda function $\lambda$ (https://en.wikipedia.org/wiki/Modular_lambda_function):
$$\lambda (5i)=\dfrac{1}{2}-6\sqrt{-360+161\sqrt{5}}.$$

So if someone can prove that
$$K(\lambda (5i))=\dfrac{\sqrt{5}+2}{20}\dfrac{\Gamma (1/4)^2}{\sqrt{\pi}}$$
and
$$E(\lambda (5i))=\dfrac{(-2+\sqrt{5})\pi^{3/2}}{\Gamma (1/4)^2}+\dfrac{\left(2+\sqrt{5}+2\sqrt{-10+6\sqrt{5}}\right)\Gamma (1/4)^2}{40\sqrt{\pi}},$$
(that's out of my reach), then Ramanujan's formula is proved.

Edit

I noticed that $K(\lambda (5i))$ can be easily proved to have the desired closed form by using division values of elliptic functions with parameter $-1$. But evaluating $E(\lambda (5i))$ seems hard…

Edit #2

The question has been answered on MathOverflow:
https://mathoverflow.net/questions/469126/how-did-ramanujan-find-sum-n-0-infty-1n-frac1-2-n1-4-n3-4-nn/469135#469135

Best Answer

Let us use the more standard notation with $k\in(0,1)$ being elliptic modulus and $k'=\sqrt{1-k^2}$ being the complementary modulus. The elliptic integrals are defined by $$K(k) =\int_0^{\pi/2}\frac {dx} {\sqrt{1-k^2\sin^2x}},E(k)=\int_0^{\pi/2}\sqrt{1-k^2\sin^2x}\,dx\tag{1}$$ The integrals $K(k), K(k'), E(k), E(k') $ are often denoted by $K, K', E, E'$ when the values $k, k'$ are available from context.

Let us now bring other players into this game and we write $q=\exp(-\pi K'/K) $ and call it the nome corresponding to modulus $k$. Ramanujan now makes use of the Dedekind eta function $$\eta(q) =q^{1/24}\prod_{n\geq 1}(1-q^n)\tag{2}$$ and its logarithmic derivative $$P(q) = 24q\frac{d}{dq}\log\eta(q)=1-24\sum_{n\geq 1}\frac{nq^n}{1-q^n}\tag{3}$$ We have the identity $$\eta(q^2)=2^{-1/3}\sqrt{\frac{2K}{\pi}}(kk')^{1/6}\tag{4}$$ which will be used in combination with the hypergeometric identity $$ \left(\frac{2K}{\pi}\right)^{2} = (1 - 2k^{2})^{-1}\,_{3}F_{2}\left(\frac{1}{4},\frac{3}{4},\frac{1}{2}; 1, 1; -\left(\frac{G^{12} - G^{-12}}{2}\right)^{-2}\right)\tag{5}$$ where $G=G(k) =(2kk') ^{-1/12}$ is one of Ramanujan's class invariant. Using these identities we get $$\eta^4(q^2)=\frac{2^{-4/3}(kk')^{2/3}}{1-2k^2} \,_{3}F_{2}\left(\frac{1}{4},\frac{3}{4},\frac{1}{2}; 1, 1; -\left(\frac{G^{12} - G^{-12}}{2}\right)^{-2}\right)\tag{6}$$ Next formidable step is to logarithmically differentiate the above identity with respect to $k$ to get a formula for $P(q^2)$. The calculations can be verified with hand and some patience and we write a few steps here. First we have $$P(q^2)=3q\frac{d}{dq}\log\eta^4(q^2)=\frac{3kk'^2}{2}\left(\frac{2K}{\pi}\right)^2\frac{d}{dk}\log\eta^4(q^2)$$ which further leads us to $$P(q^2)=\frac{3kk'^2}{2}\left(\frac{2K}{\pi}\right)^2\frac{d}{dk}\left(\log (2^{-4/3}(kk')^{2/3})+\log\left(\frac {2K}{\pi}\right) ^2\right) $$ Then we get $$P(q^2)=\left(\frac{2K}{\pi}\right) ^2(1-2k^2)+\frac{3kk'^2}{2}\frac{d}{dk}\left(\frac{2K}{\pi}\right)^2$$ Using $(5)$ we now get $$P(q^2)=\left(\frac{2K}{\pi}\right) ^2\cdot\frac{1+2k^2k'^2}{1-2k^2}+\frac{3kk'^2}{2(1-2k^2)}\frac{d}{dk}\,_{3}F_{2}\left(\frac{1}{4},\frac{3}{4},\frac{1}{2}; 1, 1; -\left(\frac{G^{12} - G^{-12}}{2}\right)^{-2}\right)$$ Switching to sigma notation we get $$P(q^2)=\frac{1+2k^2k'^2}{(1-2k^2)^2}\sum_{n\geq 0}(-1)^n\cdot\frac{(1/4)_n(1/2)_n(3/4)_n}{(n!)^3}\cdot\left(\frac{G^{12} - G^{-12}}{2}\right)^{-2n}+\frac{3kk'^2}{2(1-2k^2)}\cdot\frac{f'(k)}{f(k)}\sum_{n\geq 0}(-1)^n\cdot\frac{n(1/4)_n(1/2)_n(3/4)_n}{(n!)^3}\left(\frac{G^{12} - G^{-12}}{2}\right)^{-2n}$$ where $$f(k)= \left(\frac{G^{12} - G^{-12}}{2}\right)^{-2}$$ We have $$\frac{f'(k)} {f(k)} =(\log f(k)) '=\frac{2(1-2k^2)}{kk'^2}\cdot\frac{G^{12}+G^{-12}}{G^{12}-G^{-12}}$$ Thus we finally obtain $$P(q^2)=\frac{1+2k^2k'^2}{(1-2k^2)^2}\sum_{n\geq 0}(-1)^n\cdot\frac{(1/4)_n(1/2)_n(3/4)_n}{(n!)^3}\cdot\left(\frac{G^{12} - G^{-12}}{2}\right)^{-2n}+\frac{3(G^{12}+G^{-12})}{G^{12}-G^{-12}}\sum_{n\geq 0}(-1)^n\cdot\frac{n(1/4)_n(1/2)_n(3/4)_n}{(n!)^3}\left(\frac{G^{12} - G^{-12}}{2}\right)^{-2n}\tag{7} $$ Next we put $q=e^{-5\pi}$ in above relation and corresponding values $$2k^2=1-\sqrt{1-\phi^{-24}},2k'^2=1+\sqrt{1-\phi^{-24}}, G=(4k^2k'^2)^{-1/24}=\phi$$ (where $\phi=(1+\sqrt{5})/2$ is golden ratio) and observe that $$G^{12}-G^{-12}=F_{12}\phi+F_{11}-(-F_{12}\phi+F_{13})=2F_{12}\phi-F_{12}=F_{12}\sqrt{5}=144\sqrt{5}$$ (where $F_n$ is the Fibonacci sequence) and $$G^{12}+G^{-12}=F_{11}+F_{13}=322$$ and $$\frac{1+2k^2k'^2}{(1-2k^2)^2}=\frac{2F_{24}\phi+2F_{23}+1}{2F_{24}\phi+2F_{23}-2}=\frac{30912\phi+19105}{30912\phi+19104}$$ The above ratio can be simplified (using technique in this answer) to $$\frac{120+161\sqrt{5}}{480}$$ Thus the equation $(7)$ can be rewritten as $$P(q^2)=\frac{120+161\sqrt{5}}{480}\sum_{n\geq 0}(-1)^n\cdot\frac{(1/4)_n(1/2)_n(3/4)_n}{(n!)^3}\cdot\frac{1}{25920^n}+\frac{644\sqrt{5}}{480}\sum_{n\geq 0}(-1)^n\cdot\frac{n(1/4)_n(1/2)_n(3/4)_n}{(n!)^3}\cdot\frac{1}{25920^n}\tag{8}$$ From equation $(15)$ of this answer dealing with modular equations and related identities we have $$P(q^{2})=\frac{3}{5\pi}+\left(\frac{2K}{\pi}\right) ^26(5-3\phi)\sqrt{3\phi-4}$$ or $$ P(q^{2})=\frac{3}{5\pi}+\frac{6(5-3\phi)\sqrt{3\phi-4}} {\sqrt{1-\phi^{-24}}} \sum_{n\geq 0} (-1)^n\cdot\frac{(1/4)_n(1/2)_n(3/4)_n}{(n!)^3}\cdot\frac{1}{25920^n}$$ and this simplifies to $$P(q^{2})=\frac{3}{5\pi}+\frac{120+120\sqrt{5}} {480} \sum_{n\geq 0} (-1)^n\cdot\frac{(1/4)_n(1/2)_n(3/4)_n}{(n!)^3}\cdot\frac{1}{25920^n}\tag{9}$$ Comparing equations $(8),(9)$ we get $$\frac{288\sqrt{5}}{5\pi}=\sum_{n\geq 0}(-1)^n\cdot\frac{(1/4)_n(1/2)_n(3/4)_n}{(n!)^3}\cdot\frac{41+644n}{25920^n} $$

Related Question